х
4.
1
6 11
y
-3
-2
-1
0
slope:

4.16 11y-3-2-10slope:

Answers

Answer 1

Answer:

slope = [tex]\frac{1}{5}[/tex]

Step-by-step explanation:

Calculate the slope m using the slope formula

m = [tex]\frac{y_{2}-y_{1} }{x_{2}-x_{1} }[/tex]

with (x₁, y₁ ) = (- 4, - 3) and (x₂, y₂ ) = (11, 0) ← 2 ordered pairs from the table

m = [tex]\frac{0-(-3)}{11-(-4)}[/tex] = [tex]\frac{0+3}{11+4}[/tex] = [tex]\frac{3}{15}[/tex] = [tex]\frac{1}{5}[/tex]


Related Questions

Can someone please help I will give you 30 points and brainliest if the answer is right also no links please

Answers

Answer:

x = 9

Step-by-step explanation:

Exterior Angle property of a triangle:

The Exterior Angle property states that:

"The exterior angle of a triangle(the Angle obtained by extending one of the sides of the triangle) is equal to the sum of it's interior opposite angles."

In the given triangle RST:

∠VRS is the exterior angle, and

∠RTS and ∠RST are the interior opposite angles.

By using the above listed property,

∠VRS = ∠RTS + ∠RST

(Substituting the values of of angles from the figure)

=> (9x + 2) = 51 + (3x + 5)

=> 9x + 2 = 51 + 3x + 5

isolating x

=> 9x - 3x = 51 + 5 - 2

=> 6x = 54

dividing both sides by 6

=> x = 9

Evaluate [tex]\frac{15}{k}[/tex] when k =3

Answers

if k=3 then the answer is 5

The graph of a quadratic function has y-intercept 10 and one of its x-intercepts is 1

Answers

Answer:

  c = 10

Step-by-step explanation:

For quadratic

  y = ax^2 +bx +c

the y-intercept is c.

  c = 10

_____

The y-intercept is the value of y when x=0. Using x=0 reduces the equation to ...

  y = c

7 is 75% of what number?

Answers

Answer:

9 1/3

Step-by-step explanation:

7 - 75%

x - 100%

[tex]\frac{7 * 100}{75} = 9\frac{1}{3}[/tex]

Answer:

7 is 75% of What Number? - 7 is 75% of 9.33. 100% of 9.33 is 9.33, therefore 75 percent of 9.33 equals 7.

Step-by-step explanation:

7 is 75% of What Number? - 7 is 75% of 9.33. 100% of 9.33 is 9.33, therefore 75 percent of 9.33 equals 7.

please someone solve this please asap​

Answers

Answer:

Step-by-step explanation:

12-b

13-c

14-a

15-d

16-b

17-c

18-d]

Average velocity = v – = Displacement between two points Elapsed time between two points v – = Δ x Δ t = x 2 − x 1 t 2 − t 1 . It is important to note that the average velocity is a vector and can be negative, depending on positions x 1 and x 2 .

Step-by-step explanation:

wow,1st time u have asked important question

Is the following acute obtuse or a right triangle

Answers

Answer: Obtuse

Would appreciate brainly <3

It’s going to be an obtuse triangle.

2×5, plz help, need is really bad

Answers

Answer:10

Step-by-step explanation:

Answer:

10

Step-by-step explanation:

2×5=10

Hope this helps :)

A video game has five stages. If Will beat 4/5 of the stages, what percent of the game has he finished

Answers

Answer:

75%

Step-by-step explanation:

1 = 10%

2 = 25%

3 = 50%

4 = 75%

5 = 100%

Two mechanics worked on a car. The first mechanic charged $55 per hour, and the second mechanic charged $60 per hour. The mechanics worked for a combined total of 20 hours, and together they charged a total of $1125. How long did each mechanic work?

Answers

Answer:

15 and 5 hours.

Step-by-step explanation:

Call x the time taken by the first chap, and y the second.

[tex]\left \{ {{x+y = 20 \atop {55x+60y=1125}} \right.[/tex]

Let's divide by 5 and play with the LHS of the first equation a bit:

[tex]11x+ 11y +y = 225\\11(x+y)+y= 225[/tex] we know how much x+y is ( first equation!) let's solve.

[tex]11(20) +y = 225 \rightarrow y=5[/tex] At this point it's easy to check that [tex]x=15[/tex]

$299 for 4 tires. What is the cost for one tire? Round your answer to the nearest whole number..
$70 per tire
$75 per tire
$72 per tire

Answers

$75

$299/4 = 74.75. Rounded to the nearest whole number would be $75. :)

On August 19, 2002, the ratio of stocks gaining value on the New York Stock
Exchange to stocks losing value was about 43 to 20. If 869 stocks lost value, how
many gained?
A)892
B)1953
C)1797
D)1868

Answers

Answer:

D: 1868

Step-by-step explanation:

43:20 ratio

43/20 = 2.15

869 * 2.15 = 1,868.35

round down to nearest whole number

1,868

Questlon 21 of 25
What are the vertex and x-intercepts of the graph of the function given below?
- 2x - 24

Answers

[tex] - 2 \times - 24 = - 2( \times + 12[/tex])

(х + 5) + (х – 2)
3
7
оооо
2х + 3
2x +7
What’s the correct answer?

Answers

Answer:

the correct answer is 2x+3

Step-by-step explanation:

because your adding the (x) factors together then your adding a negitive to a positive so you would get a lower number

The picture shows $40.50 that two friends earned mowing a lawn.
They want to share the money equally.

Drag the money below to show how much each friend will get.

Answers

Answer:

Each friend gets $20.25

40.50/2=$20.25

So $20.25 each

PARALLEL PERPENDICULAR OR NEITHER

Answers

Answer:

Parallel

Step-by-step explanation:

Using these points, we can determine that the equation is as follows:

Line B: y = -2/3x + 7/3

And since Line A has the same slope, it would make Line A's equations like this:

Line A: y = -2/3x - 11/3

This makes these equations Parallel to each other because they have the same slope with different y-intercepts.

I need help with this and thought if any of you guys can help

Answers

Answer:

not enough info .

Step-by-step explanation:

sorry it needs more info

A car travels 60 miles per hour. What is the total distance the car will travel in 60 minutes?

Answers

Answer:

The car will drive 60 miles in 60 minutes (an hour)

Step-by-step explanation:

For every hour driven (60 minutes) the person will drive 60 miles, therefore it will travel a distance of 60 miles in 60 minutes

Renata moved to her new home a few years ago. Back then, the young oak tree in her back yard was 190 centimeters tall. She measured it once a year and found that it grew at a constant rate. 3 years after she moved into the house, the tree was 274 centimeters tall
How fast did the tree grow?

How many years passed from the time Renata moved in until the tree was 344 centimeters tall?

Answers

1.

The equation for this problem would be

y = (?)x + 190

With y being the height after a given amount of years and x being the amount of years passing. We would need to find ?, the rate of how fast the tree grows. The problem gives us y = 274 centimeters and x = 3 years

Let’s plug in and solve for ?

274 = ?(3) + 190

84 = ?3

28 = ?

The tree grew 28 cm per year

2.

Since we know the rate of change and y, let’s just plug in values

344 = 28x + 190

154 = 28x

5.5 = x

5.5 years since Renata moved.

Answer:

Answers below

Step-by-step explanation:

How fast did the tree grow?

28 centimeters per year

How many years passed from the time Renata moved in until the tree was 344 centimeters tall?

5.5 years

Hope this helps!

is 2(x+8)-3(y-7) linear or non linear?

Answers

Si.  es linea .U na ecuación lineal es una igualdad matemática entre dos expresiones algebraicas, denominadas miembros, en las que aparecen elementos conocidos y desconocidos (denominados variables), y que involucra solamente sumas y restas de una variable a la primera potencia.l

A rectangular pipe flowing full has the following cross-sectional dimensions Width: 6 m Height: 9 m Find the pipe's hydraulic radius in meters

Answers

45 meters unless the pipe is 60 seconds with out water

Find the 66th term of the arithmetic sequence -10, 7, 24, ...−10,7,24,...

Answers

Answer:

a(66) = -10 + 17(65) = 1095

Step-by-step explanation:

We know that this is an arithmetic sequence.  Subtracting -10 from 7 results in 17; subtracting 7 from 24 also results in 17.  Thus, the common difference, d, is 17.  The first term is -10.

The general form of an arithmetic sequence is a(n) = a(1) + d(n - 1).

Substituting -10 for a(1) and 17 for d, we get:

a(n) = -10 + 17(n - 1)

Now let n = 66:  a(66) = -10 + 17(66 - 1), or    a(66) = -10 + 17(65) = 1095

Are (1,1.5) and (12,4) solutions to the equation y=1/4x+5/4

Answers

Answer:

The first set is a solution, the second is not.

Step-by-step explanation:

Testing by filling in the numbers into the equation:

(1, 1.5): x=1, y=1.5

⇒ 1.5 =? [tex]\frac{1}{4}[/tex] · 1 + [tex]\frac{5}{4}[/tex]

⇒ 1.5 =? [tex]\frac{1}{4}[/tex] + [tex]\frac{5}{4}[/tex]

⇒ 1.5 =? [tex]\frac{6}{4}[/tex]

⇒ 1.5 =? 1.5  ⇒ Correct: is indeed solution to the equation

(12, 4): x=12, y=4

⇒ 4 =? [tex]\frac{1}{4}[/tex] · 12 + [tex]\frac{5}{4}[/tex]

⇒ 4 =? [tex]\frac{12}{4}[/tex]  + [tex]\frac{5}{4}[/tex]

⇒ 4 =? [tex]\frac{17}{4}[/tex] ⇒ Incorrect: this is not a solution for the equation

Un pilón de riego está lleno en sus cuatro séptimas partes y contiene 1 600 m3 de agua. ¿Cuántos metros cúbicos caben en el pilón?

Answers

Veremos que el volumen total que cabe en el pilón es 2,800 m^3

Sabemos que el pilón tiene una capacidad máxima V.

Tambíen sabemos que de momento, el pilón tiene 1,600 m^3 de agua, y esto es equivalente a 4/7 de la capacidad máxima.

Entonces podemos plantear:

(4/7)*V = 1,600m^3

Podemos resolver esta ecuación para V si multiplicamos ambos lados por (7/4), así obtenemos:

V = (7/4)*1,600m^3 = 2,800 m^3

Es decir, el volumen total que cabe en el pilon es 2,800m^3

Sí quieres aprender más, puedes leer:

https://brainly.com/question/19129107

A helicopter floats 120m above a helipad, a dog is 85m from the helipad, if the dog could fly, how far would it have to fly to get to helicopter?

Answers

If the dog could fly, it would need to fly 147 meters to catch the helicopter

This is a rather easy and straightforward question. We are going to solve it using Pythagoras theorem. We can interpret the question to be in a triangle where the height of the helicopter above the helipad and away from the dog is the right angle. This also means that the distance needed, between the dog and the helicopter is the hypotenuse.

Using pythagoras theorem, we have

120² + 85² = c²

c² = 14400 + 7225

c² = 21625

c = [tex]\sqrt{21625}[/tex]

c = ±147 meters.

Since distance cant be in the negative, our answer is 147 m

read more about pythagoras theorem here https://brainly.com/question/15138986

The product of two positive numbers is 9. The reciprocal of one of these numbers is 4 times the reciprocal of the other number. What is the sum of the two numbers?

Answers

Answer:

7 and 1 / 2

Step-by-step explanation:

xy = 9

( 1 / x ) = 4 ( 1 / y )

y = 4x

x ( 4x ) = 9

4x^2 = 9

x = 3 / 2

( 3 / 2 ) y = 9

y = 6

6 + ( 3 / 2 ) = 7 and 1 / 2

The two numbers are 3/2 and 6.

What are reciprocal numbers?

The reciprocal of a number is this fraction flipped upside down.

Given that, The product of two positive numbers is 9. The reciprocal of one of these numbers is 4 times the reciprocal of the other number.

Let the numbers be a and b.

ab = 9.......(i)

1/a = 4/b

a = 4b..........(ii)

Using (i) in (ii)

4b×b = 9

b² = 9/4

b = 3/2

a×3/2 = 9

a = 6

Hence, The two numbers are 3/2 and 6.

For more references on reciprocal numbers, click;

https://brainly.com/question/15590281

#SPJ5

Someone plz help me :(

Answers

Answer:

B

Step-by-step explanation:

I only help you because you have nezuko as profile picture

what is the value of m in the figure below?

Answers

Answer:  8.2 (choice B)

==========================================================

Explanation:

First we'll need the length of segment BD, which I'll call x for now. The similar triangles allow us to set up the proportion below to solve for x

AD/BD = BD/DC

13/x = x/4

13*4 = x^2

52 = x^2

x^2 = 52

x = sqrt(52)

Segment BD is exactly sqrt(52) units long.

------------

Now focus your attention on triangle BDC. We'll use the pythagorean theorem to find BC

(BD)^2 + (DC)^2 = (BC)^2

( sqrt(52) )^2 + ( 4 )^2 = ( m )^2

52 + 16 = m^2

68 = m^2

m^2 = 68

m = sqrt(68)

m = 8.2462 approximately

m = 8.2

Slove system of equation.

2x - 6y = 8
-3x + 9y = 12

Answers

answers:

2x - 6y = 8 -> x = 4 + 3y

-3x + 9y = 12 -> x = -4 + 3y

A 20-foot ladder is placed against a vertical wall of a building, with the bottom of the ladder standing on level ground 6 feet from the base of the building. How high up the wall does the ladder reach?

Answers

Answer:

19.1 feet

Step-by-step explanation:

This forms a right triangle

use the pythagorean theorem

a^2 + b^2 = c^2

a^2 + 6^2 = 20^2

a^2 + 36 = 400

a^2 = 364

a = 19.0787840283

Rounded

19.1 feet

Suppose you write the set {1000, 1001, 1002,..., 2000} in base 10. Then you record this set in base 5 and find the number in the new set whose digits have the least sum. What is this number, written in decimal notation? This number is

Answers

Decimal notation is a way of expressing a number in base ten but with a decimal part. Therefore, from the given question;

The number required is 1500The number in decimal notation is 1500.0

Number bases is a method of expressing numbers in different notations with respect to the required base. When given numbers in base ten, it is required to divide it by the number base for conversion using long division method so as to convert it to any required base. So converting each element in the given set to base 5, we have:

{1000, 1001, 1002, 1003, 1004,......, 2000}

So that:

i. 1000 converted to base 5, we have:

5  1000

5   200  r 0

5     40   r 0

5     08   r 0

5      01   r 3

       00  r 1

⇒ [tex]13000_{5}[/tex]

ii. 1001 converted to base 5, we have:

5  1001

5   200  r 1

5     40   r 0

5     08   r 0

5      01   r 3

       00  r 1

⇒ [tex]13001_{5}[/tex]

iii. 1002 converted to base 5, we have:

5  1002

5   200  r 2

5     40   r 0

5     08   r 0

5      01   r 3

       00  r 1

⇒ [tex]13002_{5}[/tex]

So then, the set in base 5 is:

{13000, 13001, 13002, 13003, 13004,......, 31000}

To determine the number whose digits have the least sum;

a. 13000 = 1 + 3 + 0 + 0 + 0 = 4

b. 13001 = 1 + 3 + 0 + 0 + 1 = 5

c. 13002  = 1 + 3 + 0 + 0 + 2 = 6

d. 13003 = 1 + 3 + 0 + 0 + 3 = 7

e. 13004 = 1 + 3 + 0 + 0 + 4 = 8

.        .        .      .    .       .           .

.        .        .      .    .       .           .

z. 31000 = 3 + 1 + 0 + 0 + 0 = 4

Thus it would be observed that in base five, 1000, 1500 and 2000 digits gave the least sum of 4, so then:

[tex]\frac{1000+1500+2000}{3}[/tex]  = 1500

The number is 1500The number in decimal notation is 1500.0

Visit: https://brainly.com/question/25112081

Answer:

I think its 1500

Step-by-step explanation:

1500

Other Questions
13. Find the distance between (-9, -1) and (-2,-6). Round to the nearest tenth.A) 8.6B) 12.1C) 9.9D) 13.0 can some one help me fill in the blank space cmo se celebra el Da Nacional del Merengue? Find the value of Y - triangle measurmentsPLEASE HELP! I WILL GIVE BRAINLIEST! brainstorm, what are treaties? What is he wanted for? sam houston Which event is caused by gravity1.Water enters an aquifer through infiltration2. Water exits a person through perspiration 3. Water changes from ice to liquid water through melting4.Water moves from the atmosphere to a glass surface through condensation Nga's car insurance premium increased by $60, which was 8% of the original cost. What was the original cost of premium? Which cabinet position was added as a result of westward expansion? A. Secretary of Transportation B. Secretary of the Interior C. Secretary of Commerce D. Secretary of Labor Multiply or divide. write fractions in simplest form. 2/3 * (-2 1/4) * (-1 1/2)i really need help with this problem! i will mark you brainliest :) a man earned 80.60 for an eight-hour day. how much would he earn at the same rate for a 38-hour week why were the middle colonies called the breadbasket What is the simplified form of the fraction below?30*O A.OB.C.oloD. 5 900 rounded to the nearest thousand Which form of the verb ALLER fits correctly in the following sentence?" Ellesweekend."au cinma leO A. vaOB. vaisO C. vontO D. allons A herniated disc is causing a patient feeling discomfort and weakness in the tibialis muscle and a possible sensation of pins and needles in the foot. at which level is the herniation exerting its pressure? Write an essay about integrity using Cyrano as either a good or bad example. Also use a personal experience or other ideas in todays society. (175 to 300 words) You are choosing between two health clubs Club A offers membership for a fee of $21 plus a monthly fee of $20. Club B offersmembership for a fee of $35 plus a monthly fee of $13. After how many months will the total cost of each health club be the same? 23 x 5 + 6 x 50 -7 = ? a dance club had 3 times as many members as an art club. The art club has 3598 fewer members than the dance club. How many members did the art club have? show bar graph